LSAT and Law School Admissions Forum

Get expert LSAT preparation and law school admissions advice from PowerScore Test Preparation.

User avatar
 Dave Killoran
PowerScore Staff
  • PowerScore Staff
  • Posts: 5850
  • Joined: Mar 25, 2011
|
#94120
Complete Question Explanation
(The complete setup for this game can be found here: lsat/viewtopic.php?f=391&p=94117)

The correct answer choice is (C)

If K is scheduled for Wednesday, then from the fourth rule V must be scheduled for Tuesday. Consequently, only H and I are available to be scheduled for Monday.

If H is scheduled for Monday, then N must be scheduled for Friday, forcing I to be scheduled on Thursday:

G2-Q9-d1.png

If I is scheduled for Monday, then H and N rotate between Thursday and Friday

G2-Q9-d2.png

The above two diagrams encompass all of the possibilities for this question, and in combination prove that answer choice (C) is correct.
User avatar
 erikariasss
  • Posts: 4
  • Joined: Feb 09, 2021
|
#87143
Hello,

I had trouble seeing why number 9 is C. It was a 'could be true' but I'm not sure why (A) or (D) are wrong answer choices.
User avatar
 Dave Killoran
PowerScore Staff
  • PowerScore Staff
  • Posts: 5850
  • Joined: Mar 25, 2011
|
#87144
Hello!

In #9, the question stem establishes that K is scheduled for Wednesday. So, from the last rule, V must be scheduled for Tuesday, which would look something like this:

  • ___ _V_ _K_ ___ ___

With answer choice (A), a tricky chain reaction occurs that causes an issue so let's look at that more closely. I is first placed on Friday from the answer condition:

  • ___ _V_ _K_ ___ _I_

Now there is only Monday or Thursday left. From the second rule, if H goes on Monday then N has to go on Friday, which can't happen since I is now there. So, H would have to go on Thursday:

  • ___ _V_ _K_ _H_ _I_

So far, so good it would seem. But, that leaves N to be scheduled on Monday (the last remaining spot), and that can't happen from the first rule. So, we can't get a viable solution here with (A), and it can be eliminated.

With (D), we now have an N :longline: I sequence. For N to be scheduled earlier than I, it would have to be on Monday or Thursday. But wait, from the first rule N can't be scheduled for Monday, which forces N into Thursday, and consequently I into Friday:

  • ___ _V_ _K_ _N_ _I_

That now leaves H to be scheduled on Monday (the last remaining spot), and from the second rule when that happens then N is scheduled for Friday, but N can't do that since it is already on Thursday. So again, no viable solution exists with the condition in (D).

A tough one since it ends up having more steps that you'd expect from a five-variable linear game!

Please let me know if that helps, thanks!
User avatar
 erikariasss
  • Posts: 4
  • Joined: Feb 09, 2021
|
#87277
That helps a lot, thank you!

I guess my next question would be clarification as to why (C) is the correct answer.
 Adam Tyson
PowerScore Staff
  • PowerScore Staff
  • Posts: 5153
  • Joined: Apr 14, 2011
|
#87301
Because in this situation, N COULD be earlier than H, erikariasss.

With VK on Tuesday and Wednesday, only H or I can go on Monday (because N cannot per the first rule). If H is Monday, N is Friday (the second rule) and I must be Thursday:

HVKIN

But when I goes on Monday, H and N become interchangeable on Thursday and Friday:

I V K H/N N/H

So in that case, it could be true that N is before H: IVKNH would work. It doesn't have to be that way, but it could be, and that's all that matters!

Get the most out of your LSAT Prep Plus subscription.

Analyze and track your performance with our Testing and Analytics Package.